K
Khách

Hãy nhập câu hỏi của bạn vào đây, nếu là tài khoản VIP, bạn sẽ được ưu tiên trả lời.

2 tháng 1 2018

Ta có:

\(21b+\frac{3}{a}=\frac{3}{a}+\frac{a}{3}+\frac{62a}{3}\ge2\sqrt{\frac{3}{a}.\frac{a}{3}}+\frac{62.3}{3}=2+62=64\left(a\ge3\right)\left(1\right)\)

Dấu "=" xảy ra \(\Leftrightarrow\frac{3}{a}=\frac{a}{3}\)và  \(a=3\Leftrightarrow a=3\)

\(\frac{21}{b}+3b=\frac{21}{b}+\frac{7b}{3}+\frac{2b}{3}\ge2\sqrt{\frac{21}{b}.\frac{7b}{3}}+\frac{2.3}{3}=14+2=16\left(b\ge3\right)\left(2\right)\)

Dấu "=" xảy ra \(\Leftrightarrow\frac{21}{b}=\frac{7b}{3}\)và  \(b=3\Leftrightarrow b=3\)

Từ (1) và (2) suy ra điều cần chứng minh.

Dấu "=" xảy ra \(\Leftrightarrow a=b=3\)

1 tháng 5 2017

Vì nó thik thì nó \(\ge\) thôi

Đúng 100%

Đúng 100%

Đúng 100%

22 tháng 1 2020

Áp dụng BĐT Cô-si cho 3 số dương ta có:

\(\left(1+\frac{1}{a}\right)^4+\left(1+\frac{1}{b}\right)^4+\left(1+\frac{1}{c}\right)^4\ge3\left(\sqrt[3]{\left(1+\frac{1}{a}\right)\left(1+\frac{1}{b}\right)\left(1+\frac{1}{c}\right)}\right)^4\)

Ta chứng minh: \(\left(1+\frac{1}{a}\right)\left(1+\frac{1}{b}\right)\left(1+\frac{1}{c}\right)\ge\left(1+\frac{3}{2+abc}\right)^3\left(1\right)\)

Theo BĐT Cô - si ta có:

\(\left(1+\frac{1}{a}\right)\left(1+\frac{1}{b}\right)\left(1+\frac{1}{c}\right)=1+\frac{1}{a}+\frac{1}{b}+\frac{1}{c}+\frac{1}{ab}+\frac{1}{bc}+\frac{1}{ca}+\frac{1}{abc}\)

\(\ge1+\frac{3}{\sqrt[3]{abc}}+\frac{3}{\sqrt[3]{\left(abc\right)^2}}+\frac{1}{abc}=\left(1+\frac{1}{\sqrt[3]{abc}}\right)^3\ge\left(1+\frac{3}{2+abc}\right)^3\)

(Vì \(abc+2=abc+1+1\ge3\sqrt[3]{abc}\))

Vậy \(\left(1\right)\) được chứng minh \(\Rightarrow BĐT\) đúng \(\forall a,b,c>0\)

Đẳng thức xảy ra \(\Leftrightarrow a=b=c=1\)

22 tháng 1 2020

Áp dụng bất đẳng thức Cauchy - Schwarz 

\(\Rightarrow VT\ge3\sqrt[3]{\left[\left(1+\frac{1}{a}\right)\left(1+\frac{1}{b}\right)\left(1+\frac{1}{c}\right)\right]^4}\)

\(\Rightarrow VT\ge3\left(\sqrt[3]{1+\frac{1}{a}+\frac{1}{b}+\frac{1}{c}+\frac{1}{ab}+\frac{1}{bc}+\frac{1}{ca}+\frac{1}{abc}}\right)^4\left(1\right)\)

Áp dụng bất đẳng thức Cauchy - Schwarz

\(\Rightarrow\hept{\begin{cases}\frac{1}{a}+\frac{1}{b}+\frac{1}{c}\ge3\sqrt[3]{\frac{1}{abc}}\\\frac{1}{ab}+\frac{1}{bc}+\frac{1}{ca}\ge3\sqrt[3]{\frac{1}{a^2b^2c^2}}\end{cases}}\)

\(\Rightarrow1+\frac{1}{a}+\frac{1}{b}+\frac{1}{c}+\frac{1}{ab}+\frac{1}{bc}+\frac{1}{ca}+\frac{1}{abc}\ge1+3\sqrt[3]{\frac{1}{abc}}\)

\(+3\sqrt[3]{\frac{1}{a^2b^2c^2}}+\frac{1}{abc}\)

\(\Rightarrow1+\frac{1}{a}+\frac{1}{b}+\frac{1}{c}+\frac{1}{ab}+\frac{1}{bc}+\frac{1}{ca}+\frac{1}{abc}\ge\left(1+\frac{1}{\sqrt[3]{abc}}\right)^3\)

\(\Rightarrow3\left(\sqrt[3]{1+\frac{1}{a}+\frac{1}{b}+\frac{1}{c}+\frac{1}{ab}+\frac{1}{bc}+\frac{1}{ca}+\frac{1}{abc}}\right)^4\)

\(\ge3\left(1+\frac{1}{\sqrt[3]{abc}}\right)^4\)

\(\left(2\right)\)

Áp dụng bất đẳng thức Cauchy - Schwarz 

\(\Rightarrow\sqrt[3]{abc}\le\frac{abc+1+1}{3}=\frac{abc+2}{3}\)

\(\Rightarrow1+\frac{1}{\sqrt[3]{abc}}\ge1+\frac{3}{abc+2}\)

\(\Rightarrow3\left(1+\frac{1}{\sqrt[3]{abc}}\right)^4\ge3\left(1+\frac{3}{abc+2}\right)^4\left(3\right)\)

Từ (1) , (2) và (3) 

\(\Rightarrow VT\ge3\left(1+\frac{3}{abc+2}\right)^4\)

\(\Leftrightarrow\left(1+\frac{1}{a}\right)^4+\left(1+\frac{1}{b}\right)^4+\left(1+\frac{1}{c}\right)^4\ge3\left(1+\frac{3}{2+abc}\right)^4\left(đpcm\right)\)

Chúc bạn học tốt !!!

28 tháng 12 2020

Đề sai. Nếu chỗ căn vế phải mà là căn bậc 3 thì t sol cho

8 tháng 7 2016

bài 2 thì bạn áp dụng bdt cô si với lựa chọn điểm rơi  hoặc bdt holder  ( nó giống kiểu bunhia ngược ) . bai 1 thi ap dung cai nay \(\frac{1}{x}+\frac{1}{y}>=\frac{1}{x+y}\)  câu 1 khó hơn nhưng bạn biết lựa chọn điểm rơi với áp dụng bdt phụ kia là ok .

9 tháng 7 2016

Bài 1:Đặt VT=A

Dùng BĐT \(\left(x+y+z\right)\left(\frac{1}{x}+\frac{1}{y}+\frac{1}{z}\right)\ge9\Rightarrow\frac{1}{x+y+z}\le\frac{1}{9}\left(\frac{1}{x}+\frac{1}{y}+\frac{1}{z}\right)x,y,z>0\)

Áp dụng vào bài toán trên với x=a+c;y=b+a;z=2b ta có:

\(\frac{ab}{a+3b+2c}=\frac{ab}{\left(a+c\right)+\left(b+c\right)+2b}\le\frac{ab}{9}\left(\frac{1}{a+c}+\frac{1}{b+c}+\frac{1}{2b}\right)\)

Tương tự với 2 cái còn lại

\(A\le\frac{1}{9}\left(\frac{bc+ac}{a+b}+\frac{bc+ab}{a+c}+\frac{ab+ac}{b+c}\right)+\frac{1}{18}\left(a+b+c\right)\)

\(\Rightarrow A\le\frac{1}{9}\left(a+b+c\right)+\frac{1}{18}\left(a+b+c\right)=\frac{a+b+c}{6}\)

Đẳng thức xảy ra khi a=b=c 

Bài 2:

Biến đổi BPT \(4\left(\frac{a^3}{\left(1+b\right)\left(1+c\right)}+\frac{b^3}{\left(1+c\right)\left(1+a\right)}+\frac{c^3}{\left(1+a\right)\left(1+b\right)}\right)\ge3\)

\(\Rightarrow\frac{a^3}{\left(1+b\right)\left(1+c\right)}+\frac{b^3}{\left(1+c\right)\left(1+a\right)}+\frac{c^3}{\left(1+a\right)\left(1+b\right)}\ge\frac{3}{4}\)

Dự đoán điểm rơi xảy ra khi a=b=c=1

\(\frac{a^3}{\left(1+b\right)\left(1+c\right)}+\frac{1+b}{8}+\frac{1+c}{8}\ge\frac{3a}{4}\)

Tương tự suy ra

\(VT\ge\frac{2\left(a+b+c\right)-3}{4}\ge\frac{2\cdot3\sqrt{abc}-3}{4}=\frac{3}{4}\)

27 tháng 11 2017

Áp dụng BĐT AM-GM ta có: 

\(VT=a^2+b^2+\frac{a}{b}+\frac{b}{a}+\frac{1}{a}+\frac{1}{b}+a+b\)

\(=1+\frac{a}{b}+\frac{b}{a}+\frac{1}{a}+\frac{1}{b}+a+b\)

\(=1+\left(\frac{a}{b}+\frac{b}{a}\right)+\left(\frac{1}{a}+2a\right)+\left(\frac{1}{b}+2b\right)-\left(a+b\right)\)

\(\ge3+2\sqrt{\frac{1}{a}\cdot2a}+2\sqrt{\frac{1}{b}\cdot2b}-\sqrt{2\left(a^2+b^2\right)}\)

\(\ge3+4\sqrt{2}-\sqrt{2}=3+3\sqrt{2}=3\left(1+\sqrt{2}\right)\)

Khi \(a=b=\frac{1}{\sqrt{2}}\) 

2 tháng 9 2015

Bài hay quá!

Theo bất đẳng thức Cô-Si cho 3 số dương ta có

\(\left(1+\frac{1}{a}\right)^4+\left(1+\frac{1}{b}\right)^4+\left(1+\frac{1}{c}\right)^4\ge3\sqrt[3]{\left(1+\frac{1}{a}\right)^4\left(1+\frac{1}{b}\right)^4\left(1+\frac{1}{c}\right)^4}\).

Do đó ta chỉ cần chứng minh \(\left(1+\frac{1}{a}\right)\left(1+\frac{1}{b}\right)\left(1+\frac{1}{c}\right)\ge\left(1+\frac{3}{2+abc}\right)^3.\) (Lúc đó kết hợp hai bất đẳng thức ta được ngay điều phải chứng minh).

Thực vậy, đầu tiên áp dụng bất đẳng thức Cô-Si cho 3 số dương ta có

\(\left(1+\frac{1}{a}\right)\left(1+\frac{1}{b}\right)\left(1+\frac{1}{c}\right)=1+\left(\frac{1}{a}+\frac{1}{b}+\frac{1}{c}\right)+\left(\frac{1}{ab}+\frac{1}{bc}+\frac{1}{ca}\right)+\frac{1}{abc}\ge\)

\(\ge1+\frac{3}{\sqrt[3]{abc}}+\frac{3}{\sqrt[3]{a^2b^2c^2}}+\frac{1}{abc}=\left(1+\frac{1}{\sqrt[3]{abc}}\right)^3.\)

Mặt khác ta có \(2+abc=1+1+abc\ge3\sqrt[3]{abc}\to\frac{1}{\sqrt[3]{abc}}\ge\frac{3}{2+abc}\to\)

\(\left(1+\frac{1}{a}\right)\left(1+\frac{1}{b}\right)\left(1+\frac{1}{c}\right)\ge\left(1+\frac{3}{2+abc}\right)^3.\)    (ĐPCM)

26 tháng 8 2017

    1. Phương pháp 1: ( Hình 1)

        Nếu  thì ba điểm A; B; C thẳng hàng.

    2. Phương pháp 2: ( Hình 2)

        Nếu AB // a và AC // a thì ba điểm A; B; C thẳng hàng.

       (Cơ sở của phương pháp này là: tiên đề Ơ – Clit- tiết 8- hình 7)

    3. Phương pháp 3: ( Hình 3)

        Nếu AB  a ; AC  A thì ba điểm A; B; C thẳng hàng.

        ( Cơ sở của phương pháp này là: Có một và chỉ một đường thẳng

        a đi qua điểm O và vuông góc với đường thẳng a cho trước

        - tiết 3 hình học 7)

        Hoặc A; B; C cùng thuộc một đường trung trực của một

        đoạn thẳng .(tiết 3- hình 7)

    4. Phương pháp 4: ( Hình 4)

        Nếu tia OA và tia OB là hai tia phân giác của góc xOy

        thì ba điểm O; A; B thẳng hàng.

        Cơ sở của phương pháp này là:                                                        

        Mỗi góc có một và chỉ một tia phân giác .

     * Hoặc : Hai tia OA và OB cùng nằm trên nửa mặt phẳng bờ chứa tia Ox ,

                   thì ba điểm O, A, B thẳng hàng.

    5. Nếu K là trung điểm BD, K là giao điểm của BD và AC. Nếu K

       Là trung điểm BD  thì K  K thì A, K, C thẳng hàng.

      (Cơ sở của phương pháp này là: Mỗi đoạn thẳng chỉ có một trung điểm)

     

C. Các ví dụ minh họa cho tùng phương pháp:

                                                                Phương pháp 1

    Ví dụ 1. Cho tam giác ABC vuông ở A, M là trung điểm AC. Kẻ tia Cx vuông góc CA

                     (tia Cx và điểm B ở hai nửa mặt phẳng đối nhau bờ AC). Trên tia Cx lấy điểm

                     D sao cho CD = AB.

                     Chứng minh ba điểm B, M, D thẳng hàng.

     Gợi ý: Muốn B, M, D thẳng hàng cần chứng minh

               Do nên cần chứng minh

BÀI GIẢI:

               AMB và CMD có:                                                       

                   AB = DC (gt).

                  

                    MA = MC (M là trung điểm AC)                                              

               Do đó: AMB = CMD (c.g.c). Suy ra:

               Mà   (kề bù) nên .

               Vậy ba điểm B; M; D thẳng hàng.

    Ví dụ 2. Cho tam giác ABC. Trên tia đối của AB lấy điểm D mà  AD = AB, trên tia đối

                     tia AC lấy điểm E mà AE = AC. Gọi M; N lần lượt là các điểm trên BC và ED

                      sao cho CM = EN.

                    Chứng minh ba điểm M; A; N thẳng hàng.

Gợi ý: Chứng minh  từ đó suy ra ba điểm M; A; N thẳng hàng.

BÀI GIẢI (Sơ lược)

          ABC = ADE (c.g.c)

          ACM = AEN (c.g.c)

          Mà  (vì ba điểm E; A; C thẳng hàng) nên

Vậy ba điểm M; A; N thẳng hàng (đpcm)

BÀI TẬP THỰC HÀNH CHO PHƯƠNG PHÁP 1

Bài 1: Cho tam giác ABC. Trên tia đối của tia AB lấy điểm D sao cho AD = AC, trên tia đối

          của tia AC lấy điểm E sao cho AE = AB. Gọi M, N lần lượt là trung điểm của BE và

          CD.

          Chứng minh ba điểm M, A, N thẳng hàng.

Bài 2: Cho tam giác ABC vuông ở A có . Vẽ tia Cx  BC (tia Cx và điểm A ở

          phía ở cùng phía bờ BC), trên tia Cx lấy điểm E sao cho CE = CA. Trên tia đối của tia

          BC lấy điểm F sao cho BF = BA.

          Chứng minh ba điểm E, A, F thẳng hàng.

Bài 3: Cho tam giác ABC cân tại A, điểm D thuộc cạnh AB. Trên tia đối của tia CA lấy điểm

          E sao cho CE = BD. Kẻ DH và EK vuông góc với BC (H và K thuộc đường thẳng BC)

          Gọi M là trung điểm HK.

          Chứng minh ba điểm D, M, E thẳng hàng.

Bài 4: Gọi O là trung điểm của đoạn thẳng AB. Trên hai nửa mặt phẳng đối nhau bờ AB, kẻ

          Hai tia Ax và By sao cho .Trên Ax lấy hai điểm C và E(E nằm giữa A và C),

          trên By lấy hai điểm D và F ( F nằm giữa B và D) sao cho AC = BD, AE = BF.

          Chứng minh ba điểm C, O, D thẳng hàng , ba điểm E, O, F thẳng hàng.

Bài 5.Cho tam giác ABC . Qua A vẽ đường thẳng xy // BC. Từ điểm M trên cạnh BC, vẽ các

          đường thẳng song song AB và AC, các đường thẳng này cắt xy theo thứ tự tại D và E.

          Chứng minh các đường thẳng AM, BD, CE cùng đi qua một điểm.

                                                              PHƯƠNG PHÁP 2

    Ví dụ 1: Cho tam giác ABC. Gọi M, N lần lượt là trung điểm của các cạnh AC, AB. Trên

                  Các đường thẳng BM và CN lần lượt lấy các điểm D và E sao cho M là trung  

                 điểm BD và N là trung điểm EC.

                  Chứng minh ba điểm E, A, D thẳng hàng.

Hướng dẫn: Xử dụng phương pháp 2                                            

                  Ta chứng minh AD // BC và AE // BC.

BÀI GIẢI.

                 BMC và DMA có:

                   MC = MA (do M là trung điểm AC)

                    (hai góc đối đỉnh)

                   MB = MD (do M là trung điểm BD)

                  Vậy: BMC = DMA (c.g.c)

                   Suy ra: , hai góc này ở vị trí so le trong nên BC // AD (1)

                   Chứng minh tương tự : BC // AE (2)

                   Điểm A ở ngoài BC có một và chỉ một đường thẳng song song BC nên từ (1)

                   và (2) và theo Tiên đề Ơ-Clit suy ra ba điểm E, A, D thẳng hàng. 

   Ví dụ 2: Cho hai đoạn thẳng  AC và BD cắt nhau tai trung điểm O của mỗi đoạn. Trên tia

                 AB lấy lấy điểm M sao cho B là trung điểm AM, trên tia AD lấy điểm N sao cho

                 D là trung điểm AN.